Đến nội dung

Hình ảnh

Tổng hợp các bài BĐT

ẩn đi không cho xem

  • Please log in to reply
Chủ đề này có 164 trả lời

#121
tr2512

tr2512

    Thượng sĩ

  • Thành viên
  • 272 Bài viết

Hâm nóng lại topic chút  :D

Cho các số thực dương a, b, c. Chứng minh rằng:

${\left( {\sqrt {\frac{a}{{b + c}}} + \sqrt {\frac{b}{{a + c}}} + \sqrt {\frac{c}{{a + b}}} } \right)^2} + \frac{1}{6}\left( {\frac{{{a^2}}}{{{b^2}}} + \frac{{{b^2}}}{{{c^2}}} + \frac{{{c^2}}}{{{a^2}}}} \right) \ge 5$

                                                       -Tự sáng tác-



#122
canletgo

canletgo

    Sĩ quan

  • Thành viên
  • 389 Bài viết

Chứng minh rằng: $n!>(\frac{n}{3})^{n}$ với $\forall n\in\mathbb{N}^*$


Alpha $\alpha$ 


#123
toanhoc2017

toanhoc2017

    Thiếu úy

  • Banned
  • 628 Bài viết
Bài của bạn Lengocthuc195:Bạn cố gắng học gõ Telex đi nhé. Cũng dễ AK bạn. Cho $a,b,c$ là các số thực dương thỏa mãn $a+b+c=3$. Chứng minh rằng
$A=\sum\frac{a+b}{\sqrt{2a+b}+\sqrt{2b+a}}\leq\sqrt{3}$. Mình (Trương Huỳnh Nhật Vinh) giúp gõ lại bài rồi

Bài viết đã được chỉnh sửa nội dung bởi toanhoc2017: 30-07-2018 - 10:53


#124
kiemthecao

kiemthecao

    Lính mới

  • Thành viên mới
  • 2 Bài viết

Nhiều dạng hay quá ạ



#125
Agon Dise

Agon Dise

    Binh nhì

  • Thành viên mới
  • 13 Bài viết

Cho a,b,c > 0 . Chứng minh rằng:
$\frac{a^2}{b}+\frac{b^2}{c}+\frac{c^2}{a}\geq \sqrt[]{a^2-ab+b^2}+\sqrt[]{b^2-bc+c^2}+\sqrt[]{c^2-ca+a^2}$


Do your best


#126
Agon Dise

Agon Dise

    Binh nhì

  • Thành viên mới
  • 13 Bài viết

Mình giải bài của mình luôn
Ta có $\frac{a^2}{b}+\frac{b^2}{c}+\frac{c^2}{a}\geq a+b+c$

=> $2(\frac{a^2}{b}+\frac{b^2}{c}+\frac{c^2}{a})\geq \frac{a^2}{b}+\frac{b^2}{c}+\frac{c^2}{a} + a + b + c$

 

=> VT $\geq (\frac{a^2-ab+b^2}{b}+b)+(\frac{b^2-bc+c^2}{c}+c)+(\frac{c^2-ca+a^2}{a}+a)$

=> VT $\geq 2\sqrt{a^2-ab+b^2} + 2\sqrt{b^2-bc+c^2} +2\sqrt{c^2-ca+a^2}$

Ta có điều phải chứng minh


Bài viết đã được chỉnh sửa nội dung bởi Agon Dise: 19-10-2018 - 22:53

Do your best


#127
Agon Dise

Agon Dise

    Binh nhì

  • Thành viên mới
  • 13 Bài viết

Cho a,b,c $\geq0$ và a + b + c = 1. Chứng minh:
$a+\sqrt{ab}+\sqrt[3]{abc}\leq \frac{4}{3}$


Do your best


#128
Agon Dise

Agon Dise

    Binh nhì

  • Thành viên mới
  • 13 Bài viết

Cho a,b,c $\geq0$ và a + b + c = 1. Chứng minh:
$a+\sqrt{ab}+\sqrt[3]{abc}\leq \frac{4}{3}$

Ta có $a+\sqrt{ab}+\sqrt[3]{abc}\leq \frac{4}{3}$ = $a + \sqrt{\frac{a}{2}.2b} +\sqrt[3]{\frac{a}{4}.b.4c}\leq a+\frac{a}4+b+\frac{a}{12}+\frac{b}{3}+\frac{4c}{3}=\frac{4}{3}(a+b+c)=\frac{4}{3}$


Bài viết đã được chỉnh sửa nội dung bởi Agon Dise: 20-10-2018 - 21:13

Do your best


#129
Agon Dise

Agon Dise

    Binh nhì

  • Thành viên mới
  • 13 Bài viết

Cho a,b,c > 0 . Chứng minh rằng:

 $\frac{\sqrt{ab}}{a+b+2c}+\frac{\sqrt{bc}}{b+c+2a}+\frac{\sqrt{ca}}{c+a+2b}\leq \frac{3}{4}$


Do your best


#130
Agon Dise

Agon Dise

    Binh nhì

  • Thành viên mới
  • 13 Bài viết

Cho a,b,c > 0 . Chứng minh rằng:

 $\frac{\sqrt{ab}}{a+b+2c}+\frac{\sqrt{bc}}{b+c+2a}+\frac{\sqrt{ca}}{c+a+2b}\leq \frac{3}{4}$

Ta có $\sum \frac{4\sqrt{ab}}{a+b+2c}\leq \sum \frac{(\sqrt{a}+\sqrt{b})^{2}}  {a+b+2c}\leq \sum\frac{a}{a+c}+\frac{b}{b+c}=3$ 

Ta có điều phải chứng minh


Do your best


#131
LoveMath1234567

LoveMath1234567

    Binh nhất

  • Thành viên mới
  • 22 Bài viết

 

Topic này chủ yếu là coppy nội dung đề bài và lời giải của các mem lại, các ĐHV THPT có thể coppy tiếp CD13 nhưng nên để chung trong một khung. Tính đúng sai của các lời giải thì kiểm tra lại sau.

 

Bắt đầu vậy!

 

Bài 1:

Cho abc=1 va $a^{3}> 36.  CMR  :\frac{a^{2}}{3}+b^{2}+c^{2}> ab +bc+ca$}

Lời giải:

$VT-VP=\frac{a^{2}}{4}+b^{2}+c^{2}-ab-bc+2bc+\frac{a^{2}}{12}=(\frac{a}{2}-b-c)^{2}+\frac{a^{2}-36bc}{12}>0\Rightarrow$ đpcm

Cách khác:

Từ giả thiết suy ra $a>0$ và $bc>0$. Bất đẳng thức cần chứng minh tương đương với
\[\dfrac{a^2}{3}+(b+c)^2-3bc-a(b+c)\ge 0\\ \iff \dfrac{1}{3}+\left(\dfrac{b+c}{a}\right)^2-\dfrac{b+c}{a}-\dfrac{3}{a^3}\ge 0\]
Vì $a^3>36$ nên \[\dfrac{1}{3}+\left(\dfrac{b+c}{a}\right)^2-\dfrac{b+c}{a}-\dfrac{3}{a^3}> \left(\dfrac{b+c}{a}\right)^2-\dfrac{b+c}{a}+ \dfrac{1}{4}= \left(\dfrac{b+c}{a}-\dfrac{1}{2}\right)^2 >0\]
 
 

 

Bài 2:

Với a,b,c >0; n ∈ N*.CMR:

$\frac{a^{n}}{b+c}+\frac{b^{n}}{a+c}+\frac{c^{n}}{a+b}\geq \frac{3}{2}\left ( \frac{a^{n}+b^{n}+c^{n}}{a+b+c} \right )$

Lời giải:

$\sum \frac{a^{n}}{b+c}\geq \frac{1}{3}(\sum a^{n})(\sum \frac{1}{a+b})\geq \frac{1}{3}(\sum a^{n})(\frac{9}{2(a+b+c)})=\frac{3}{2}(\frac{\sum a^{n}}{\sum a})$

 

 

 

Bài 3:

Cho $x,y,z >0$ thỏa điều kiện $x^{2}+y^{2}+z^{2}=9$

Tìm giá trị nhỏ nhất của $P=\frac{x^5}{y^2}+\frac{y^5}{z^2}+\frac{z^5}{x^2}$

Lời giải:

Theo $Cauchy$ Ta có:

$$\dfrac{x^5}{y^2}+\dfrac{x^5}{y^2}+\sqrt{3}y^2+\sqrt{3}y^2+3\sqrt{3}\ge \sqrt{3}x^2$$

Cách khác:

Sử dụng Cauchy-Schwarzt ta có 

       $\frac{x^5}{y^2}+\frac{y^5}{z^2}+\frac{z^5}{x^2}\geqslant \frac{(x^3+y^3+z^3)^2}{xy^2+yz^2+zx^2}$

Sử dụng Cauchy-Schwarzt và AM-GM ta có 

       $xy^2+yz^2+zx^2\leqslant \sqrt{(x^2+y^2+z^2)(x^2y^2+y^2z^2+z^2x^2)}\leqslant \sqrt{\frac{(x^2+y^2+z^2)^3}{3}}=3$

Do đó $P\geqslant \frac{(x^3+y^3+z^3)^2}{3}\geqslant \frac{(x^2+y^2+z^2)^3}{9}=3$ 

Đẳng thức xảy ra khi $x=y=z=1$

 

 

 

Bài 4:

Ch0 $a>0$ và $n$ là 1 số tự nhiên

Chứng minh rằng $a^n+\frac{1}{a^n}-2\geqslant n^2(a+\frac{1}{a}-2)$

Lời giải:

Bất đẳng thức tương đương với $(a^{n-1}+a^{n-2}+...+a+1)\geq n^2a^{n-1}$ (hiển nhiên theo AM-GM)

Cách khác:

Do tính đối xứng giữa a và $\frac{1}{a}$ nên ta có thể giả sử a ≥ 1.  đặt $\sqrt{a}$ =x ≥ 1.bdt $\Leftrightarrow$ $x^{2n}+\frac{1}{x^{2n}}-2 \geq n^{2}(x^{2}+\frac{1}{x^{2}}-2)\Leftrightarrow (x^{n}-\frac{1}{x^{n}})^{2}\geq n^{2}(x-\frac{1}{x})^{2} \Leftrightarrow $x^{n}-\frac{1}{x^{n}}\geq n(x-\frac{1}{x})$①.

Với x=1 thì ① đúng

Với x>1 thì ① $\Leftrightarrow x^{n-1} +x^{n-3} ...+\frac{1}{x^{n-3}}+\frac{1}{x^{n-1}}\geq n$ (đúng vì theo bđt AM-GM).

Dấu bằng xảy ra khi x=1 $\Leftrightarrow a=1$

 

 

 

Bài 5:

Cho $a,b,c,d$ là các số thực thỏa mãn $\left\{\begin{matrix} a+b+c+d=0\\a^2+b^2+c^2+d^2=2 \end{matrix}\right.$

Tìm GTLN của $P=abcd$

Lời giải:

Áp dụng AM-GM ta có 

$2=\sum a^{2}\geq 4\sqrt[4]{\prod a^{2}}\Rightarrow \sqrt{\left | abcd \right |}\leq \frac{1}{2}\Rightarrow abcd\leq \frac{1}{4}$

Dấu bằng xảy ra khi $a=b=-c=-d=\frac{1}{\sqrt{2}}$ và các hoán vị của chúng

 

 

 

Bài 6:

Cho $a,\,b,\,c\geq 0$ thỏa mãn $a+b+c=1.$ Tìm giá trị lớn nhất của biểu thức: $$P=abc\left(a^2+b^2+c^2\right)$$

Lời giải:

Ta có: $P=abc(a+b+c)(a^{2}+b^{2}+c^{2})\leq \frac{1}{3}(ab+bc+ca)^{2}(a^{2}+b^{2}+c^{2})$

 

Mặt khác, lại có: $(ab+bc+ca)^{2}(a^{2}+b^{2}+c^{2})\leq \left (\frac{(a+b+c)^{2}}{3} \right )^{3}=\frac{1}{27}$

 

Do đó: $P\leq \frac{1}{81}$

 

Dấu bằng xảy ra khi và chỉ khi $a=b=c= \frac{1}{3}$

 

 

 

Bài 7:

Cho các số thực $x,\,y>0$ thỏa mãn $3x+y\leq1.$ Tìm giá trị nhỏ nhất của biểu thức: $$S=\dfrac{1}{x}+\dfrac{1}{\sqrt{xy}}$$

Lời giải:

$S\geq \frac{1}{x}+\frac{1}{\sqrt{x(1-3x)}}$

$\geq \frac{1}{x}+\frac{2}{1-2x}=\frac{2}{x(1-x)}\geq \frac{8}{(x+1-x)^{2}}=8$

Dấu "=" xảy ra $\Leftrightarrow x=\frac{1}{4}$

 

 

 

Bài 8:

Cho các số thực a,b,c,x,y thỏa mãn $$ax-by=\sqrt{3}$$ .

Tìm GTNN của $F= a^{2}+b^{2}+x^{2}+y^{2}+ bx +ay$

Lời giải:

Sử dụng giả thiết $ax-by=\sqrt{3}$ ta có:
$$(a^2+b^2)(x^2+y^2)=(ax+by)^2+(ax-by)^2=(ax+by)^2+3$$
Áp dụng bất đẳng thức $Cauchy$ , suy ra:
$$a^2+b^2=x^2+y^2=(a^2+b^2)+(x^2+y^2) \\ \ge 2\sqrt{(a^2+b^2)(x^2+y^2)}=2\sqrt{(ax+by)^2+3}$$
Do đó, ta đưa về bài toán tìm GTNN của: $2\sqrt{x^2+3}+x$ trong đó $x=ax+by$
Ta có:
$$\left(2\sqrt{x^2+3}+x\right)^2=4(x^2+3)+4x\sqrt{x^2+3}+x^2 \\ = (x^2+3)+4x\sqrt{x^2+3}+4x^2+9 \\ = \left(\sqrt{x^2+3}+2x\right)^2+9\ge 9$$
$$\Rightarrow 2\sqrt{x^2+3}+x\ge 3$$
Vậy $\text{MinT}=\fbox{3}$
 
 
 

Bài 9:

Cho các số thực dương $a,b,c$. Tìm giá trị nhỏ nhất của biểu thức:

 

                                $P=\frac{2}{a+ \sqrt{ab}+ \sqrt[3]{abc}}-\frac{3}{\sqrt{a+b+c}}$

Lời giải:

$a+\sqrt{\frac{1}{2}a.2b}+\sqrt[3]{\frac{1}{4}a.b.4c}\leq a+\frac{1}{4}a+b+\frac{1}{12}a+\frac{1}{3}b+\frac{4}{3}c=\frac{4}{3}(a+b+c)$

Do đó $P\geq \frac{3}{2(a+b+c)}-\frac{3}{\sqrt{a+b+c}}$...

 

 

 

Bài 10:

Cho x,y là các số không âm thoả $x^{3}+y^{3}\leq 1$

 

Tìm giá trị lớn nhất của $P=2\sqrt{x}+\sqrt{y}$

Lời giải:

$(x^3+y^3)(\sqrt[5]{2^6}+1)^5\geqslant (2\sqrt{x}+\sqrt{y})^6$

$\Leftrightarrow 2\sqrt{x}+\sqrt{y}\leqslant \sqrt[6]{(\sqrt[5]{2^6}+1)^5}$

Vậy $Max(P)= \sqrt[6]{(\sqrt[5]{2^6}+1)^5}\Leftrightarrow \frac{a^3}{2\sqrt[5]{2}}=b^3=\frac{1}{2\sqrt[5]{2}+1}$

 

Chiều coppy tiếp, sau đó kiểm tra nội dung sau.

 

 

Bài 11:Cho các số a,b,c không âm không đồng thời bằng không. Chứng minh rằng;

$\sum \frac{2a^{2}-bc}{b^{2}-bc+c^{2}}\geq 3$

 

Lời giải:(vutuanhien)

 

Không mất tính tổng quát, ta có thể giả sử $b$ là số nằm giữa $a$ và $c$

BĐT đã cho tương đương với

$$\sum \frac{2a^2+(b-c)^2}{b^2-bc+c^2}\geq 6$$

Áp dụng BĐT Cauchy-Schwarz, ta có

$$\sum \frac{2a^2}{b^2-bc+c^2}\geq \frac{2(a^2+b^2+c^2)^2}{\sum a^2(b^2-bc+c^2)}=\frac{2(a^2+b^2+c^2)^2}{2\sum a^2b^2-abc\sum a}$$

$$\sum \frac{(b-c)^2}{b^2-bc+c^2}\geq \frac{[a(b-c)+b(a-c)+c(a-b)]^2}{2\sum a^2b^2-abc\sum a}=\frac{4b^2(a-c)^2}{2\sum a^2b^2-abc\sum a}$$

Do đó ta chỉ cần chứng minh

$$(a^2+b^2+c^2)^2+2b^2(a-c)^2\geq 6\sum a^2b^2-3abc\sum a (1)$$

Ta có 

$b^2(a-c)^2=[a(b-c)+c(a-b)]^2=a^2(b-c)^2+c^2(a-b)^2+2ac(a-b)(b-c)$

$\geq a^2(b-c)^2+c^2(a-b)^2$

Suy ra 

$$2b^2(a-c)^2\geq a^2(b-c)^2+b^2(c-a)^2+c^2(a-b)^2$$

$$\Rightarrow VT (1)\geq (\sum a^2)^2+2\sum a^2b^2-2abc\sum a$$

Do đó ta chỉ còn phải chứng minh 

$$(\sum a^2)^2+2\sum a^2b^2-2abc\sum a\geq 6\sum a^2b^2-3abc\sum a$$

$$\Leftrightarrow \sum a^4+abc\sum a\geq 2\sum a^2b^2$$

BĐT này hiển nhiên đúng theo BĐT Schur

$$\sum a^4+abc\sum a\geq \sum ab(a^2+b^2)$$

Và BĐT AM-GM

$$\sum ab(a^2+b^2)\geq 2\sum a^2b^2$$

Kết thúc chứng minh 

Đẳng thức xảy ra khi $a=b=c$ hoặc $a=b$, $c=0$ và các hoán vị.

 

Bài 12:(bosulan239)

Cho a,b,c là các số không âm không đồng thời bằng không.

CMR

$\frac{\sum a^{2}}{\sum ab}\geq \sum \frac{ab}{b^{2}+bc+c^{2}}$

Bài giải:(vutuanhien)

 

BĐT đã cho tương đương với

$\frac{a^2}{ab+bc+ca}-\frac{ab}{b^2+bc+c^2}+\frac{b^2}{ab+bc+ca}-\frac{bc}{c^2+ca+a^2}+\frac{c^2}{ab+bc+ca}-\frac{ca}{a^2+ab+b^2}\geq 0$

$\Leftrightarrow \sum \frac{ac(ac-b^2)}{b^2+bc+c^2}\geq 0$

Do $\frac{ac(ac-b^2)}{b^2+bc+c^2}=\frac{ac^2(a+b+c)}{b^2+bc+c^2}-ac$ nên BĐT đã cho có thể viết lại thành

$\sum \frac{ac^2(a+b+c)}{b^2+bc+c^2}\geq ab+bc+ca$

$\Leftrightarrow \sum \frac{ac^2}{b^2+bc+c^2}\geq \frac{ab+bc+ca}{a+b+c}$

Áp dụng BĐT Cauchy-Schwarz, ta có

$VT\geq \frac{(ab+bc+ca)^2}{\sum a(b^2+bc+c^2)}=\frac{ab+bc+ca}{a+b+c}$

Kết thúc chứng minh 

 

 


Bài 13: (nguyencuong123)

Cho a,b,c không âm thoả mãn: $a+b+c=3$

Chứng Minh: $\sum \frac{a+1}{ab+1}\geq 3$

 

 

Bài giải:(Juliel)

Áp dụng AM-GM cho vế trái, ta cần chứng minh :

$(a+1)(b+1)(c+1)\geq (ab+1)(bc+1)(ca+1)\Leftrightarrow abc+(ab+bc+ca)+(a+b+c)+1\geq a^{2}b^{2}c^{2}+abc(a+b+c)+(ab+bc+ca)+1\Leftrightarrow abc+4\geq a^{2}b^{2}c^{2}+3abc+1\Leftrightarrow a^{2}b^{2}c^{2}+2abc\leq 3$

Hiển nhiên đúng vì $abc\leq (\frac{a+b+c}{3})^{3}=1$

 


 


Bài 14:(Chrome98):Chứng minh bất đẳng thức sau với $a,b,c>0$ và $a+b+c=1$:

 

\[ \frac{a^2}{3a+1}+\frac{b^2}{3b+1}+\frac{c^2}{3c+1}\ge 24\left(\frac{a^2}{9a+1}+\frac{b^2}{9b+1}+\frac{c^2}{9c+1}\right)^2 \]

Bài giải: (Simpson Joe Donald)

 

 
 
$\bullet\ AM-GM:\ \dfrac{a^2}{9a+1}= \dfrac{a^2}{6a+(3a+1)}\le \dfrac{a^2}{2\sqrt{6a(3a+1)}}= \dfrac{a\sqrt{a}}{2\sqrt{6(3a+1)}}$ ;
$\bullet\ Cauchy-Schwarz:\ VP\le \left( \dfrac{a\sqrt{a}}{\sqrt{3a+1}}+  \dfrac{b\sqrt{b}}{\sqrt{3b+1}}+  \dfrac{c\sqrt{c}}{\sqrt{3c+1}}\right)^2\le (a+b+c).VT=VT$

 

Bài 15:(trauvang97:)Cho các số thực dương $a,b,c$ thoả mãn:

 

                             $\frac{2}{a^{2}+1}+\frac{2}{b^{2}+1}+\frac{2}{c^{2}+1}\geq 3$

 

Chứng minh rằng: $(a-2)^{2}+(b-2)^{2}+(c-2)^{2}\geq 3$

 

Bài giải:

 

(Nguyen Huy Tuyen)$\frac{2}{a^{2}+1}+\frac{2}{b^{2}+1}+\frac{2}{c^{2}+1}\geq 3\Leftrightarrow \sum \frac{(1-a)(1+a)}{a^2+1}\geqslant 0$

$(a-2)^{2}+(b-2)^{2}+(c-2)^{2}-3=\sum (a-3)(a-1)$

Ta có :$\sum (a-3)(a-1)-\sum \frac{2(1-a)(1+a)}{a^2+1}=\sum \frac{(a-1)^4}{a^2+1}\geqslant 0$

           $\Leftrightarrow \sum (a-3)(a-1)\geqslant \sum \frac{2(1-a)(1+a)}{a^2+1}\geqslant 0$

           $\Leftrightarrow (a-2)^{2}+(b-2)^{2}+(c-2)^{2}\geq 3$

 

 

Bài 16:(phanquockhanh)Cho $x,y,z >0 : xyz+x+z=y$ . Tìm giá trị lớn nhất của biểu thức : $P=\frac{2}{x^2+1} - \frac{2}{y^2+1} -\frac{4z}{\sqrt{z^2+1}}+\frac{3z}{(z^2+1).\sqrt{z^2+1}}$

(Trích đề thi thử số 2 – THTT)

Bài giải:

(trauvang97)Từ giả thiết ta có: $x=\frac{y-z}{1+yz}$.

 

Khi đó:

 

$P=\frac{2(1+yz)^{2}}{(y^{2}+1)(z^{2}+1)}-\frac{2}{y^{2}+1}-\frac{4z}{\sqrt{z^{2}+1}}+\frac{3z}{(z^{2}+1)\sqrt{z^{2}+1}}$

 

$P=\frac{2z(2y+(y^{2}-1)z)}{(y^{2}+1)(z^{2}+1)}-\frac{4z}{\sqrt{z^{2}+1}}+\frac{3z}{(z^{2}+1)\sqrt{z^{2}+1}}$

 

Do $\frac{2z(2y+(y^{2}-1)z)}{(y^{2}+1)(z^{2}+1)}=\frac{2z\sqrt{(2y+(y^{2}-1)z)^{2}}}{(y^{2}+1)(z^{2}+1)}\leq \frac{2z\sqrt{(4y^{2}+(y^{2}-1)^{2})(1+z^{2})}}{(y^{2}+1)(z^{2}+1)}=\frac{2z}{\sqrt{z^{2}+1}}$

 

Do đó:

 

$P\leq \frac{2z}{\sqrt{z^{2}+1}}-\frac{4z}{\sqrt{z^{2}+1}}+\frac{3z}{\sqrt{z^{2}+1}}\left ( 1-\frac{z^{2}}{z^{2}+1} \right )$

 

$P=-3t^{3}+t$ với $\frac{z}{\sqrt{z^{2}+1}}=t\in (0;1)$

 

Khảo sát hàm số trên ta thấy $maxP=\frac{2}{9}\Leftrightarrow x=\frac{\sqrt{2}}{2};y=\sqrt{2},z=\frac{\sqrt{2}}{4}$

 

 

 


Bài 17:(Toc Ngan)Cho $a,b,c >0$ và $a+b+c=3$

Chứng minh rằng : $8(\frac{1}{a}+\frac{1}{b}+\frac{1}{c})+9 \geq 10(a^2+b^2+c^2)$

 Bài giải:(babystudymaths)

 

Cách 1

Giải như sau:

Giả sử a là số lớn nhất trong 3 số a,b,c ,thế thì c nhỏ hơn 3 và không nhỏ hơn 1

ta thấy $9=(42a-48)+(42b-\frac{69}{2})+(42c-\frac{69}{2})$

Thay và BĐT ban đầu ta thấy tương đương

$(\frac{8}{b}-10b^{2}+42b-\frac{69}{2})+(\frac{8}{c}-10c^{2}+42c-\frac{69}{2})\geq 10a^{2}-\frac{8}{a}-42a+48\Leftrightarrow \frac{(16-5b)(2b-1)^{2}}{b}+\frac{(16-52)(2c-1)^{2}}{c}\geq \frac{4(5a-1)(a-2)^{2}}{a}$

Áp dụng BCS ,ta có:

VT $\geq \frac{(2b-1+2c-1)^{2}}{\frac{b}{16-5b}+\frac{c}{16-5c}}= \frac{4(a-2)^{2}}{\frac{b}{16-5b}+\frac{c}{16-5c}}$

Lúc này ta chỉ cần chứng minh 

$\frac{a}{5a-1}\geq \frac{b}{16-5b}+\frac{c}{16-5c}$

Mà $\frac{b}{16-5b}+\frac{c}{16-5c}\leq \frac{b}{16-5a}+\frac{c}{16-5a}= \frac{3-a}{16-5a}\leq \frac{a}{5a-1}\Leftrightarrow \frac{1}{(5a-1)(16-5a)}> 0$

ĐÚng theo giả thiết,từ đây ta suy ra đ.p.c.m

Đẳng thức xảy ra khi và chỉ khi a=b=1/2 ,c=2 cùng hoán vị

 

Cách 2: 

Ta có BĐT tương đương

f(abc,a+b+c,ab+bc+ca) =$8.\frac{ab+bc+ca}{abc}+9-10((a+b+c)^{2}-2(ab+bc+ca))\geq 0$

Nhận thấy đây là hàm đơn điệu trên R theo abc nên theo định lý ABC, hàm số đạt cực tiểu khi có 2 biến = nhau, nên a=b=$\frac{3-c}{2}$

Thay vào và chứng minh BĐT 1 biến c ,bài toán trở nên quá đơn giản 

 

 

 

 


Bài 18:(caybutbixanh)Cho $x;y;z> 0$.Chứng minh rằng :

$P=\frac{2xy}{(z+x)(z+y)}+\frac{2yz}{(x+y)(x+z)}+\frac{3xz}{(y+z)(y+x)}\geqslant \frac{5}{3}.$

 

(trích đề thi học sinh giỏi lớp 11-Quảng Bình 2011)

--------------------------------

 

 

 

(T M) Hướng giải:

 

Bằng khai triển trực tiếp ta đưa bất đẳng thức cần chứng minh thành

 

$$xy(x+y)+yz(y+z)+4xz\left ( x+z \right )\geq 10xyz$$

 

Điều này tương đương với

 

$$\frac{x+y}{z}+\frac{y+z}{x}+\frac{4(x+z)}{y}\geq 10$$

 

Áp dụng $AM-GM$ từng cặp là ra.

 

 


Bài 19:(supermath98)Cho các sô dương $a;b;c$ thỏa mãn $\large ab+ac+bc=3abc$. Tìm GTNN của biểu thức: 

 

$\large M=\frac{2\left ( a^{2}b^{2}+b^{2}c^{2}+a^{2}c^{2} \right )+abc}{a^{2}b^{2}c^{2}}$

 

 

Bài giải:(thanhdok14)

 

 

Vì $a, b, c>0$ nên từ điều kiện ban đầu, ta suy ra:

$\frac{1}{a}+\frac{1}{b}+\frac{1}{c}=3$

Đặt: $\left(\frac{1}{a};\frac{1}{b};\frac{1}{c}\right)\to (x;y;z)$

$\Rightarrow x+y+z=3$

$\Rightarrow xy+yz+zx\le 3$

Mặt khác: $M$ được viết lại thành:

$M=2\left(\frac{1}{a^2}+\frac{1}{b^2}+\frac{1}{c^2}\right)+\frac{1}{abc}$

$=2(x^2+y^2+z^2)+xyz$

Lại có: $x^2+y^2+z^2=9-2(xy+yz+zx)$

$xyz\ge \frac{(x+y+z)[4(xy+yz+zx)-(x+y+z)^2]}{9}=\frac{4(xy+yz+zx)-9}{3}$   (theo $schur$)

Từ đó ta có:

$M\ge \frac{4}{3}(xy+yz+zx)-4(xy+yz+zx)+15=\frac{-8}{3}(xy+yz+zx)+15\ge 7$   (vì $xy+yz+zx\le 3$)

Vậy $min M=7\Leftrightarrow a=b=c=1$

 

 


Bài 20:(duaconcuachua)

Cho $a,b,c$ là các số thực dương thỏa mãn $ab+bc+ca=abc$.

Chứng minh rằng $\frac{a^{4}+b^{4}}{ab(a^{3}+b^{3})}+\frac{b^{4}+c^{4}}{bc(b^{3}+c^{3})}+\frac{c^{4}+a^{4}}{ca(c^{3}+a^{3})}\geq 1$

 Bài giải:(Sagittius912)Theo bđt Chebyshev ta có

 

 

$\frac{a^4+b^4}{a^3+b^3}\ge \frac{a+b}{2}$

do đó

 

$\frac{a^{4}+b^{4}}{ab(a^{3}+b^{3})}+\frac{b^{4}+c^{4}}{bc(b^{3}+c^{3})}+\frac{c^{4}+a^{4}}{ca(c^{3}+a^{3})}\ge \frac{a+b}{2ab}+\frac{b+c}{2bc}+\frac{c+a}{2ca}=\frac{ab+bc+ca}{abc}=1$

 

Dấu đẳng thức xảy ra khi $a=b=c=3$

 

 


 

 

 

ở chỗ bài 6 làm sao mk ra dòng thứ 4 trên xuống thế ạ

 

 



#132
Kitaro1006

Kitaro1006

    Binh nhất

  • Thành viên mới
  • 27 Bài viết

a,b,c>0. a+b+c = 1                                             

$\frac{1+a}{1-a}+\frac{1+b}{1-b}+\frac{1+c}{1-c}\leqslant 2\left ( \frac{a}{b}+\frac{b}{c}+\frac{c}{a} \right )$



#133
Vbmtygo

Vbmtygo

    Lính mới

  • Thành viên mới
  • 1 Bài viết

Hâm nóng lại topic chút :D
Cho các số thực dương a, b, c. Chứng minh rằng:
${\left( {\sqrt {\frac{a}{{b + c}}} + \sqrt {\frac{b}{{a + c}}} + \sqrt {\frac{c}{{a + b}}} } \right)^2} + \frac{1}{6}\left( {\frac{{{a^2}}}{{{b^2}}} + \frac{{{b^2}}}{{{c^2}}} + \frac{{{c^2}}}{{{a^2}}}} \right) \ge 5$
-Tự sáng tác-



#134
Rhythme

Rhythme

    Lính mới

  • Thành viên mới
  • 5 Bài viết

Cho a, b, c là ba số thực không âm (a,b,c chỉ có nhiều nhất một số bằng 0)

 

Tìm giá trị nhỏ nhất của biểu thức:

 

$P= \frac{a}{b+c} + \frac{b}{c+a} + \frac{c}{a+b} + 4\sqrt{2}\sqrt{\frac{ab + bc +ca}{a^{2}+b^{2} + c^{2}}}$

 

(Mình nhẩm min=6)


Quần chúng ngốc nghếch đợi người thông não Ợ_Ợ


#135
Bolshevik

Bolshevik

    Binh nhì

  • Thành viên mới
  • 17 Bài viết

$\frac{a^{2}}{b}+\frac{b^{2}}{c}+\frac{c^{2}}{a}$



#136
Doflamingo

Doflamingo

    Hạ sĩ

  • Thành viên
  • 82 Bài viết
Cho x, y, z>0 và x+y+z=3xyz. CM:

$\frac{y^{2}}{x^{3}(3y^{2}+1)}+\frac{z^{2}}{y^{3}(3z^{2}+1)}+\frac{x^{2}}{z^{3}(3x^{2}+1)}\geq \frac{3}{4}$



#137
CD13

CD13

    Thượng úy

  • Thành viên
  • 1456 Bài viết

File 55 bài. Lâu quá không tham gia diễn đàn, quên cả cách post bài.

Cách đính kèm file chỗ nào nhỉ?


Bài viết đã được chỉnh sửa nội dung bởi CD13: 18-03-2019 - 15:58


#138
Unrruly Kid

Unrruly Kid

    Trung sĩ

  • Thành viên
  • 113 Bài viết

Bây giờ không thấy ai online cũng không ai up bài nữa :(


Đôi khi ngươi phải đau đớn để nhận thức, vấp ngã để trưởng thành, mất mát để có được, bởi bài học lớn nhất của cuộc đời được dạy bằng nỗi đau.

#139
DOTOANNANG

DOTOANNANG

    Đại úy

  • ĐHV Toán Cao cấp
  • 1609 Bài viết

Chứng minh với $\left \vert \it{c} \right \vert\leqq \it{b}$$:$

$$\left ( \frac{\it{21}\,\it{a}^{\,\it{3}}- \it{17}\,\it{a}\it{b}^{\,\it{2}}- \it{4}\,\it{c}^{\,\it{3}}}{\it{21}} \right )^{\,\it{2}}\geqq \it{(}\,\,\it{a}^{\,\it{2}}- \it{b}^{\,\it{2}}\,\,\it{)}^{\,\it{3}}$$

 



#140
DOTOANNANG

DOTOANNANG

    Đại úy

  • ĐHV Toán Cao cấp
  • 1609 Bài viết

Chứng minh với $\it{a},\,\it{b},\,\it{c}> \it{0}$ sao cho $\it{a}+ \it{b}+ \it{c}= \it{constant}$$:$

$$\it{constant}\leqq \min\,\{\,\,\it{a}^{\,\it{2}}\it{(}\,\,\it{cb}+ \it{a}\,\,\it{)},\,\it{b}^{\,\it{2}}\it{(}\,\,\it{ac}+ \it{b}\,\,\it{)},\,\it{c}^{\,\it{2}}\it{(}\,\,\it{ba}+ \it{c}\,\,\it{)}\,\,\}\leqq \it{constant}$$






1 người đang xem chủ đề

0 thành viên, 1 khách, 0 thành viên ẩn danh